1) k1 = 5.0 * 10-3;
T1 = 215 oC = 488 K.
2) k2 = 1.2 * 10-1 (let us assume that the constant has such a value since, by the condition of the task, it is not completely clear);
T2 = 452 oC = 725 K.
3) k3 = ???
T3 = 100 oC = 373 K.
Solution (a):
Arrhenius equation looks like this:
π = π΄πβπΈπ/π π
Where:
π is the rate constant, π΄ is the preβexponential factor, πΈπ is the activation energy, π is the ideal gas constant (8.314 π½βπΎ β πππ), and π is the absolute temperature of the system.
Then, for two points Arrhenius equation looks like this:
ln(π2/π1) = πΈπ*(1/π1 β 1/π2 ) / R
Then,
ln(1.2 * 10-1 / 5.0 * 10-3) = πΈπ * (1/488 β 1/725 ) / 8.314;
ln(24) = πΈπ * 8.057 * 10-5;
3.178 = πΈπ * 8.057 * 10-5;
πΈπ = 39444.63 π½βπππ = 39.44 kπ½βπππ.
Answer (a): πΈπ = 39.44 kπ½βπππ.
Solution (b):
ln(π2/π3) = πΈπ*(1/π3 β 1/π2 ) / R
Then,
ln(1.2 * 10-1/π3) = 39444.63 * (1/373 β 1/725 ) / 8.314;
ln(1.2 * 10-1/π3) = 6.1755;
ln(1.2 * 10-1) - ln(π3) = 6.1755;
-2.12026 - ln(π3) = 6.1755;
ln(π3) = -8.29576;
π3 = e-8.29576 = 2.49 * 10-4;
π3 = 2.49 * 10-4.
Check:
ln(π1/π3) = πΈπ*(1/π3 β 1/π1 ) / R
Then,
ln(5.0 * 10-3/π3) = 39444.63 * (1/373 β 1/488 ) / 8.314;
ln(5.0 * 10-3/π3) = 2.9974;
ln(5.0 * 10-3) - ln(π3) = 2.9974;
-5.2983 - ln(π3) = 2.9974;
ln(π3) = -8.2957;
π3 = e-8.2957 = 2.49 * 10-4;
π3 = 2.49 * 10-4. - Correct.
Answer (b): π3 = 2.49 * 10-4.
Answer: (a) πΈπ = 39.44 kπ½βπππ; (b): π3 (at 100 oC) = 2.49 * 10-4.
Comments
Leave a comment